Equilibrium Style Question involving a Weight

In summary, the problem involves finding the mass and traction force of a weight supporting a fractured tibia using a variation of the Russel traction apparatus. The weight of the leg and foot is 51.5N and two equations are used to solve for the traction force and mass. The different angles in the problem may result in a net upward force, which can be used to find the mass.
  • #1
Emethyst
118
0

Homework Statement


While a fractured tibia is healing, it must be kept horizontal and under some tension. One method of doing this is to support the leg by using a variation of the Russel traction apparatus. The lower leg (including the foot) of a patient weighs 51.5N, which must be supported in the traction apparatus. a) What is the mass of the weight shown? b) What traction force does the apparatus apply along the direction of the leg?


Homework Equations


Newton's second law, pythagorean's theorem, right triangle properties



The Attempt at a Solution


No idea where to start with this. The picture of the situation is provided (if it shows up, hopefully). My idea was to find two equations for the traction force and then sub one into the other to solve for the weight. My first equation was tan46=FN/Fapp, where FN=FG leg + Weight and tan37=weight/Fapp. I think this is wrong though because I get an answer that seems too large for the weight, so I have no idea how to properly go about this question. If anyone can be of assistance it would be greatly appreciated, thanks in advance.
 

Attachments

  • UNTITLED1.PNG
    UNTITLED1.PNG
    2 KB · Views: 515
Physics news on Phys.org
  • #2
Weird. I approved the attachment, and can see a small thumbnail of it, but when I click on it, I only get a black image with a few words on it. Can anybody else expand the thumbnail?

Emethyst, can you maybe repost the attachment as a PDF? PrimoPDF is a free writer if you don't have one already.
 
  • #3
Ok does this work for the attachment?
 

Attachments

  • UNTITLED1.pdf
    10 KB · Views: 238
  • #4
I can see it now. I'm not sure, but I think the fact that the two angles are different will give you a net upward force at the leg, which must equal the weight of the leg and foot. That should let you work back to the mass.
 

1. What is equilibrium in relation to a weight?

Equilibrium is a state in which all forces acting on an object are balanced, resulting in no net movement or acceleration of the object. In the case of a weight, equilibrium occurs when the downward force of gravity is balanced by an equal and opposite force, such as tension or support from another object.

2. How is equilibrium achieved for a weight?

Equilibrium for a weight can be achieved by adjusting the position or strength of the opposing forces acting on the weight. This can be done by changing the length or angle of a supporting structure, or by adding or removing weight from the opposing force.

3. What factors affect equilibrium for a weight?

The factors that affect equilibrium for a weight include the weight of the object, the strength and direction of the forces acting on the weight, and the stability and strength of the supporting structures. Any changes to these factors can alter the equilibrium state of the weight.

4. How does the center of mass play a role in equilibrium for a weight?

The center of mass, or the point at which all of the weight of an object is concentrated, is crucial in determining equilibrium for a weight. When the center of mass is directly above or below the point of support, the weight is in equilibrium. However, if the center of mass is shifted to one side, the weight will become unbalanced and may tip over.

5. Can equilibrium for a weight be maintained indefinitely?

In theory, equilibrium for a weight can be maintained indefinitely if the opposing forces remain constant and there are no external factors that disrupt the balance. However, in reality, there are always small fluctuations and changes that can affect the equilibrium state, so it may not be possible to maintain it indefinitely.

Similar threads

  • Introductory Physics Homework Help
Replies
1
Views
7K
  • Introductory Physics Homework Help
Replies
1
Views
6K
  • Introductory Physics Homework Help
Replies
7
Views
5K
  • Introductory Physics Homework Help
Replies
3
Views
6K
  • Introductory Physics Homework Help
Replies
1
Views
2K
  • Introductory Physics Homework Help
Replies
5
Views
4K
  • Introductory Physics Homework Help
Replies
14
Views
3K
  • Introductory Physics Homework Help
Replies
7
Views
6K
  • Introductory Physics Homework Help
Replies
13
Views
2K
  • Introductory Physics Homework Help
Replies
1
Views
1K
Back
Top